OB Exam 1

Pataasin ang iyong marka sa homework at exams ngayon gamit ang Quizwiz!

A female client is concerned about the rate of fertility because the client is older than age 35 years. The client states, "Have I given up the chance to be a parent?" Which response from the nurse is accurate?

"A woman's fertility peaks in the 20s, declines gradually until age 35, and then rapidly declines until it ends." Explanation: The nurse is correct to identify that pregnancy for this woman may be more difficult. The client's fertility relates to the client's age. A woman's fertility begins at menarche, peaks during the 20s, and then declines gradually until age 35. After age 35, the decline is more precipitous until menopause, when fertility ends. Women in their 40s and beyond often require reproductive assistance to successfully become pregnant.

The school nurse is presenting a lecture to adolescents to teach them how conception occurs. Which statement by the nurse would accurately describe this process?

"Conception usually occurs when the ovum is in the outer third of the fallopian tube."

A client with diabetes is in the first trimester of pregnancy and is currently having difficulty keeping blood glucose levels within normal limits. The client explains that she has been "eating for two" so the baby is healthy. How should the nurse respond to the client?

"Elevated blood glucose levels in the first trimester have been linked to congenital anomalies."

A client with diabetes is in the first trimester of pregnancy and is currently having difficulty keeping blood glucose levels within normal limits. The client explains that she has been "eating for two" so the baby is healthy. How should the nurse respond to the client?

"Elevated blood glucose levels in the first trimester have been linked to congenital anomalies." Explanation; The first trimester of pregnancy is the most important time for fetal development. If the client can control hyperglycemia during this time, the chances of a congenital anomaly are greatly reduced. Infants of clients with poorly controlled diabetes tend to be large. At birth, babies born to clients with uncontrolled diabetes are prone to respiratory distress syndrome. Elevated blood glucose levels do not hasten the development of the fetus in utero and can lead to hydramnios.

A nurse is working with a pregnant client to schedule follow-up visits for the pregnancy. Which statement by the client indicates that she understands the scheduling?

"From now until I am 28 weeks' pregnant, I will be coming once a month."

A pregnant client has been diagnosed with hydramnios and has been referred for further testing. To obtain family history information needed for the testing, which question(s) should the nurse ask the client? Select all that apply.

"Has anyone in your family been born with a problem with the intestines?" "Do you have anyone in the family who has been born with a spine problem?" "Do you have anyone in the family born with Down syndrome?" Explanation: Hydramnios is defined as having more than 2000 ml of amniotic fluid. It is associated with problems occurring in the mother or in the fetus. For the mother, hydramnios may be caused by diabetes. For the fetus, it may be caused by neural tube defects, chromosomal defects, or malformations of the central nervous and gastrointestinal systems. Cardiac and renal problems are not generally associated with hydramnios.

During an exam, the nurse notes that the blood pressure of a client at 22 weeks' gestation is lower, and her heart rate is 12 beats per minute higher than at her last visit. How should the nurse interpret these findings?

Both findings are normal at this point of the pregnancy.

A client with phenylketonuria (PKU) has just learned that they are pregnant. The client's most recent phenylalanine level is 10 mg/dl (605.44 mcmol/l). The nurse is providing anticipatory guidance on the expected course of pregnancy. What will the nurse include in the teaching? Select all that apply.

Clients with uncontrolled PKU can have pregnancies complicated by phenylalanine embryopathy (PE). Intellectual impairments can occur in pregnant clients with PKU that is not adequately managed. Phenylalanine levels will be checked one to two times a week with diet adjustments accordingly. Explanation: Pregnant clients with phenylketonuria (PKU) can expect to have phenylalanine levels checked one to two times per week and have their diets adjusted accordingly. Phenylalanine embryopathy (PE) is a fetal complication of uncontrolled PKU in the pregnant client. Individuals with poorly managed PKU can develop intellectual impairments from high serum levels of phenylalanine and its metabolites. Intrauterine growth restriction, not fetal macrosomia, can result from PE. The client's current phenylalanine level of 10 mg/dl (605.44 mcmol/l) is high; the goal is a level of 2 to 6 mg/dl (121.09 to 363.26 mcmol/l) during pregnancy.

A nurse is providing care to a client at 33 weeks' gestation, who is hospitalized for treatment of diabetic ketoacidosis. The client has a blood glucose level of 280 mg/dl (15.54 mmol/l), has polyuria, and is nauseous and lethargic. Which nursing intervention(s) will the nurse perform in the client's care? Select all that apply.

Conduct attentive fetal heart rate monitoring. Administer fluid volume replacement. Provide glycemic control. Administer replacement electrolytes. Explanation: Nursing interventions should prioritize efforts to control glucose levels, fluid volume and electrolyte replacement to address polyuria and nausea, and close fetal monitoring. Diabetic ketoacidosis is not an indication for an emergent birth. Any abnormal variations of the fetal heart tracing usually resolves once the pregnant client's condition improves.

A client at 28 weeks' gestation is seen during a prenatal visit. The nurse notes that the client's most recent hemoglobin level is 10.9 g/dl (109 g/l). Which is an appropriate action for the nurse to take?

Continue with routine prenatal assessments.

A nurse is teaching a group of adolescent girls about the menstrual cycle. The nurse would integrate information about which hormones as being predominant during the cycle? Select all that apply.

GnRH FSH LH

Many women develop iron-deficient anemia during pregnancy. What diagnostic criteria would the nurse monitor for to determine anemia in the pregnant woman?

Hematocrit of 32% or less

A client at 11 weeks' gestation experiences pregnancy loss. The client asks the nurse if the bleeding and cramping that occurred during the miscarriage were caused by working long hours in a stressful environment. What is the most appropriate response from the nurse?

"I can understand your need to find an answer to what caused this. Let's talk about this further." Explanation: Talking with the client may assist her to explore her feelings. She and her family may search for a cause for a spontaneous early bleeding so they can plan for future pregnancies. Even with modern technology and medical advances, however, a direct cause cannot usually be determined.

A pregnant woman is diagnosed with iron-deficiency anemia and is prescribed an iron supplement. After teaching her about the prescribed iron supplement, which statement indicates successful teaching?

"I need to drink plenty of fluids to prevent constipation."

The nurse instructs a pregnant client with sickle cell anemia on ways to prevent a crisis. Which client statement indicates that teaching has been effective?

"I should drink eight glasses of water every day." Explanation: The fluid status of a pregnant client with sickle cell anemia is important because dehydration can precipitate a crisis. The client should drink at least eight glasses of fluid each day to prevent dehydration. Clients with sickle cell anemia should not take an iron supplement because the sickled cells cannot incorporate iron in the same way as nonsickled cells. Standing for long periods of time can cause red cell destruction in the client with sickle cell anemia. The client should sit with the legs elevated to encourage venous return of blood from the lower extremities.

A pregnant woman has developed varicosities. Which statement would suggest she needs additional health teaching?

"I wear knee-highs rather than pantyhose." Explanation: Women with varicosities should not wear knee-high stockings as they put pressure on leg veins and reduce venous return.

A nurse is providing care to a client who is at 26 weeks' gestation. The client is known to have a substance use disorder with alcohol and tobacco. The client is also homeless. Which statement(s) is appropriate for the nurse to make to the client? Select all that apply.

"If you like, I can ask the social worker to talk to you about housing options." "Drinking alcohol during pregnancy can lead to many physical and developmental problems in the baby." "Many people find nicotine reduction products helpful in stopping or cutting down on smoking." Explanation:Counseling about the negative impact of substance use in pregnancy should be presented in a factual, nonjudgmental manner. Therefore, the nurse should inform the client in a matter-of-fact, nonjudgmental manner the many problems that drinking alcohol and smoking can cause in the fetus. The nurse should also inform the client of available options to help stop or reduce their smoking. The nurse should also let the client know that there are options available to them for housing if they are interested. Telling the client they should stop drinking for their and their child's health or that the child can be born prematurely and small are presented in a judgmental manner. Although these statements are true, they should be presented in a nonjudgmental manner.

The nursing instructor has completed a teaching session on the female reproductive system. The instructor determines more teaching is necessary when a student indicates which action occurs in the menstrual cycle?

"In the ischemic phase, the uterine lining becomes ischemic because of increased estrogen and progesterone."

The nurse is preparing a female client for an emergency cesarean birth at 30 weeks' gestation. The primary health care provider prescribes a glucocorticoid injection for the client. When preparing to administer this injection, the client asks what purpose the injection serves. Which response by the nurse is accurate?

"It promotes the formation of surfactant in the fetal lungs."

A nurse is teaching a pregnant woman with preterm premature rupture of membranes (PPROM) about caring for herself after she is discharged home (which is to occur later this day). Which statement by the woman indicates a need for additional teaching?

"It's okay for my husband and I to have sexual intercourse." Explanation; The woman with preterm PROM should monitor her baby's activity by performing fetal movement (kick) counts daily, check her temperature and report any increases to the health care provider, not insert anything into her vagina or vaginal area, (such as tampons or having vaginal intercourse), and avoid sitting in a tub bath.

A woman in her second trimester comes for a follow-up visit and says to the nurse, "I feel like I'm on an emotional roller-coaster." Which response by the nurse would be most appropriate?

"Mood swings are completely normal during pregnancy."

The nurse is teaching a prenatal class on potential problems during pregnancy to a group of expectant parents. The risk factors for placental abruption (abruptio placentae) are discussed. Which comment validates accurate learning by the parents?

"Placental abruption is quite painful and I will need to let the doctor know if I begin to have abdominal pain." Placental abruption (abruptio placentae) occurs when there is a spontaneous separation of the placenta from the uterine wall. It can occur anywhere on the placenta and will cause painful, dark red vaginal bleeding. If the abruption is small, the ob/gyn will try to deliver the fetus vaginally. But if severe bleeding occurs or there is fetal distress, a cesarean birth will be performed. Women older than 35 are also at higher risk for developing placental abruption.

A client in the 26th week of pregnancy is concerned because the hemoglobin level continues to decline despite following an iron-enriched diet. Which response should the nurse make to the client?

"The hemoglobin level may drop until the 28th week of pregnancy."

A client presents to the clinic for an initial prenatal visit. The client has a history of systemic lupus erythematosus (SLE). Which question is most important for the nurse to ask to determine risks to pregnancy outcomes?

"When was your last exacerbation episode?"

A student nurse is preparing for a presentation that will illustrate the various physiologic changes in the woman's body during pregnancy. Which cardiovascular changes up through the 26th week should the student point out?

Increased pulse rate and decreased blood pressure

A nurse is caring for a 45-year-old pregnant client with a cardiac disorder who has been instructed by her primary care provider to follow class I functional activity recommendations. The nurse correctly instructs the client to follow which limitations?

"You do not need to limit your physical activity unless you experience any problems such as fatigue, chest pain, or shortness of breath."

The nurse is caring for a pregnant woman in her third trimester who is HIV positive. The client voices concerns about how to prevent transmitting HIV to her infant. Which response by the nurse is most appropriate?

"You will not be able to breastfeed your infant once born. Your infant will need to receive formula."

A woman is scheduled to undergo fetal nuchal translucency testing. Which statement would the nurse include when describing this test?

"You'll have an intravaginal ultrasound to measure fluid in the fetus."

A client who has had type 1 diabetes since childhood has just given birth. The client asks the nurse if giving birth will alter the need for insulin. What response will the nurse give?

"Your insulin requirements will decrease, but will still be needed."

In teaching a fertility class, what is the most appropriate method for the nurse to explain self-ovulation monitoring that promotes conception?

14 days before the next menses is expected

A pregnant client who is planning to have genetic testing asks the nurse when she should schedule her amniocentesis. What should the nurse tell the client?

16 weeks

When a woman is planning pregnancy, what is her window around ovulation to have intercourse?

2 days before to 1 day after ovulation Explanation:At ovulation, the consistency of the mucus changes so that sperm can swim through it, allowing fertilization. At the same time, the mucus-secreting glands of the cervix actually become able to store live sperm for 2 or 3 days. These sperm can later move up through the corpus and into the fallopian tubes to fertilize the egg; thus, intercourse 1 or 2 days before ovulation can lead to pregnancy. Additionally, since the ovum lives approximately 24 hours, intercourse one day after ovulation may result in pregnancy.

A nurse is providing care to several pregnant women at different weeks of gestation. The nurse would expect to screen for group B streptococcus (GBS) infection in the client who is at:

36 weeks' gestation.

During pregnancy a woman's blood volume increases to accommodate the growing fetus to the point that vital signs may remain within normal range without showing signs of shock until the woman has lost what percentage of her blood volume?

40%

The nurse is caring for a client at the prenatal clinic. The client reports that she has felt some fluttering sensations in her lower abdomen and she noticed that her waistline is now totally gone. Additionally, she shows the nurse her nipples and the areola are much darker. Based upon this assessment, in which month of pregnancy is this client?

4th month Explanation: Based upon the presenting findings, the mother is in her 4th month of pregnancy. This is when quickening occurs—the beginning of feeling fetal movement. Also, the mother now has lost her waistline and the breast areola has begun to darken.

A client who takes levothyroxine for hypothyroidism is having an initial prenatal visit. Which teaching will the nurse provide to this client?

Blood tests will be done every 4 to 6 weeks during the first trimester to adjust the medication dose.

A client who is 16 weeks pregnant has a lower blood pressure than that of prepregnancy levels. What should the nurse realize as being the cause for this lower blood pressure?

A decrease in the second trimester may occur because of placental growth. explanation; In some women, blood pressure actually decreases slightly during the second trimester because the expanding placenta causes peripheral resistance to circulation to lower. The lower blood pressure is not because prepregnancy blood pressure measurements were inaccurate. Blood pressure does not normally decrease throughout the entire pregnancy. There is no enough information to determine if the client is dehydrated; however, this is not the reason for the blood pressure to be lower in the second trimester of pregnancy.

A nurse is providing preconception counseling to a client who has epilepsy and is taking valproate to control seizure activity. What anticipatory guidance will the nurse provide regarding the disease process and the progression of pregnancy? Select all that apply.

A higher dose of folic acid will be needed to prevent neural tube defects. Continue the recommended dose of valproate to prevent seizures. Avoid individual triggers associated with seizure activity. Expect that medication dosages may need titration as pregnancy progresses.

Hormone levels of a woman indicate that the corpus luteum stopped functioning and releasing progesterone after 5 weeks. The nurse would recognize that which scenario is the expected outcome?

A spontaneous abortion (miscarriage) would occur.

A nurse suspects that a pregnant client may be experiencing abruption placenta based on assessment of which of the following? (Select all that apply.) A) Dark red vaginal bleeding B) Insidious onset C) Absence of pain D) Rigid uterus E) Absent fetal heart tones

A) Dark red vaginal bleeding D) Rigid uterus E) Absent fetal heart tones

A nurse is teaching a pregnant woman at risk for preterm labor about what to do if she experiences signs and symptoms. The nurse determines that the teaching was successful when the woman makes which statement? A."I'll drink several glasses of water." B."I'll sit down to rest for 30 minutes." C."I'll try to move my bowels." D."I'll lie down with my legs raised."

A. "I'll drink several glasses of water."If the woman experiences any signs and symptoms of preterm labor, she should stop what she is doing and rest for 1 hour, empty her bladder, lie down on her side, drink two to three glasses of water, feel her abdomen and note the hardness of the contraction, and call her health care provider and describe the contraction

A nurse is teaching a woman about measures to prevent preterm labor in future pregnancies because the woman just experienced preterm labor with her most recent pregnancy. The nurse determines that the teaching was successful based on which statement by the woman? A."I'll make sure to limit the amount of long distance traveling I do." B."Stress isn't a problem that is related to preterm labor." C."I'll need extra iron in my diet so I have a extra for the baby." D."Separating pregnancies by about a year should be helpful."

A. "I'll make sure to limit the amount of long distance traveling I do."

With regard to protein in the diet of pregnant women, nurses should be aware that: A. Many protein-rich foods are also good sources of calcium, iron, and B vitamins. B. Many women need to increase their protein intake during pregnancy. C. As with carbohydrates and fat, no specific recommendations exist for the amount of protein in the diet. D. High-protein supplements can be used without risk by women on macrobiotic diets a

A. Many protein-rich foods are also good sources of calcium, iron, and B vitamins.

With regard to medications, herbs, shots, and other substances normally encountered, the maternity nurse should be aware that during pregnancy: A. Prescription and over-the-counter (OTC) drugs that otherwise are harmless can be made hazardous by metabolic deficiencies of the fetus. B. The greatest danger of drug-caused developmental deficits in the fetus is seen in the final trimester. C. Killed-virus vaccines (e.g., tetanus) should not be given, but live-virus vaccines (e.g., measles) are permissible. D. No convincing evidence exists that secondhand smoke is potentially dangerous to the fetus. a

A. Prescription and over-the-counter (OTC) drugs that otherwise are harmless can be made hazardous by metabolic deficiencies of the fetus.

With regard to the father's acceptance of the pregnancy and preparation for childbirth, the maternity nurse should know that: A. The father goes through three phases of acceptance of his own. B. The father's attachment to the fetus cannot be as strong as that of the mother because it does not start until after birth. C. In the last 2 months of pregnancy, most expectant fathers suddenly get very protective of their established lifestyle and resist making changes to the home. D. Typically men remain ambivalent about fatherhood right up to the birth of their child.

A. The father goes through three phases of acceptance of his own.

The nurse is reviewing the process of egg maturation and ovulation with a client. What occurs during ovulation in the ovarian cycle that the nurse should include in the teaching session with the client?

About day 14, a surge of hormones cause the ovum to burst through the ovary. Explanation; The follicular phase lasts from about day 4 to about day 14. During this time, under the influence of FSH, several follicles begin to ripen, and the ovum within each begins to mature. About day 14, a surge of hormones causes the ovum to burst through the ovary. This act is called ovulation. During the luteal phase, the empty, ruptured graafian follicle becomes the corpus luteum and begins to secrete progesterone and estrogen. The endometrium of the uterus has a similar cycle. It is called the uterine cycle or endometrial cycle. This process prepares the uterus for implantation of an ovum (egg).

A nurse is caring for a client with hyperemesis gravidarum. Which nursing action is the priority for this client?

Administer IV normal saline with vitamins and electrolytes. Explanation: The first choice for fluid replacement is generally normal saline with vitamins and electrolytes added. If the client does not improve after several days of bed rest, "gut rest," IV fluids, and antiemetics, then total parenteral nutrition or percutaneous endoscopic gastrostomy tube feeding is instituted to prevent malnutrition.

An Rh-negative client is having an amniocentesis. What is the highest priority nursing intervention to perform immediately after the procedure?

Administer Rh immune globulin. explanation; A woman with Rh-negative blood type and having an amniocentesis needs Rh immune globin after the procedure to protect against fetal isoimmunization. All women also need to be observed for 30 minutes after the procedure to check for labor contractions and that the fetal heart rate remains normal.

A client at 36 weeks' gestation is admitted to labor and delivery for induction of labor. The client has a history of poorly controlled type 2 diabetes. The client had a biophysical profile with an abnormal result that prompted the need for induction of labor. Which intervention does the nurse expect to implement during the induction process?

Administer and titrate an intravenous insulin infusion. Explanation: The nurse expects to administer and titrate an intravenous (IV) insulin infusion based on glucose levels of the pregnant client. Because the client has poor glycemic control, the client will require IV insulin during their labor induction. Glucocorticoids are administered to the pregnant client to improve fetal outcomes, if birth is anticipated prior to 34 weeks' gestation; this client is at 36 weeks' gestation. A high-risk labor and birth require continuous electronic fetal monitoring, not intermittent monitoring. Glucose levels of the pregnant client should be monitored every 2 to 6 hours and, in some cases, hourly, depending on glucose levels.

During a routine antepartal visit, a pregnant woman reports a white, thick vaginal discharge. What would the nurse do next?

Ask the woman if she is having any itching or irritation. Explanation: Although vaginal secretions increase during pregnancy, the nurse would need to ascertain if this discharge is the normal leukorrhea of pregnancy or if it is a monilial vaginitis, which is common during pregnancy. The nurse needs additional information to conclude that the woman's report is normal. A culture may or may not be necessary. There is no evidence to suggest that her membranes have ruptured.

The nurse is caring for a postpartum client with type 2 diabetes. Which intervention will the nurse add to this client's plan of care?

Assess blood glucose before and after meals.

The nurse is caring for a postpartum client with type 2 diabetes. Which intervention will the nurse add to this client's plan of care?

Assess blood glucose before and after meals. Explanation: After birth, clients with type 2 diabetes should have medications prescribed according to the glucose level, which should be completed fasting, before meals, and after meals. The postpartum client with type 1 diabetes should have their blood glucose level measured every 4 to 6 hours. Neither diabetes nor the medications used to treat it are contraindicated for breastfeeding. There is no reason for the client to bottle feed the newborn because of the condition. Within 48 hours after giving birth, the client with type 1 diabetes will require approximately half of the insulin dose that was needed while pregnant.

A nurse is caring for a client with cardiovascular disease who has just given birth. What nursing interventions should the nurse perform when caring for this client? Select all that apply.

Assess for shortness of breath. Assess for a moist cough. Assess for edema and note any pitting. Auscultate heart sounds for abnormalities.

A nurse is providing information regarding ovulation to a couple who want to have a baby. Which fact should the nurse tell the clients?

At ovulation, a mature follicle ruptures, releasing an ovum.

A pregnant woman experiencing nausea and vomiting should: A. Drink a glass of water with a fat-free carbohydrate before getting out of bed in the morning. B. Eat small, frequent meals (every 2 to 3 hours). C. Increase her intake of high-fat foods to keep the stomach full and coated. D. Limit fluid intake throughout the day. b

B. Eat small, frequent meals (every 2 to 3 hours).

A nurse teaches a pregnant woman about the presumptive, probable, and positive signs of pregnancy. The woman demonstrates understanding of the nurse's instructions if she states that a positive sign of pregnancy is: A. A positive pregnancy test result. B. Fetal movement palpated by the nurse-midwife. C. Braxton Hicks contractions. D. Quickening. b

B. Fetal movement palpated by the nurse-midwife.

A pregnant patient is experiencing some integumentary changes and is concerned that they may represent abnormal findings. The nurse provides information to the patient that the following findings would be considered "normal abnormal" findings during pregnancy so that she should not be alarmed. (Select all that apply.) A. Facial edema B. Melasma C. Linea nigra D. Superficial thrombophlebitis E. Vascular spiders F. Allodynia b,c,e

B. Melasma C. Linea nigra E. Vascular spiders

In understanding and guiding a woman through her acceptance of pregnancy, a maternity nurse should be aware that: A. Nonacceptance of the pregnancy very often equates to rejection of the child. B. Mood swings are most likely the result of worries about finances and a changed lifestyle, as well as profound hormonal changes. C. Ambivalent feelings during pregnancy are usually seen only in emotionally immature or very young mothers. D. Conflicts such as not wanting to be pregnant or childrearing and career-related decisions need not be addressed during pregnancy because they will resolve themselves naturally after birth. b

B. Mood swings are most likely the result of worries about finances and a changed lifestyle, as well as profound hormonal changes.

A nurse is teaching a group of primigravida women who are in their first trimester. One of the women asks the nurse about sexual activity during pregnancy. Which information would the nurse most likely incorporate into the response?

Because of pelvic congestion, women may experience increased clitoral sensitivity.

The nurse is reviewing the plan of care for a pregnant client experiencing a threatened miscarriage. Which outcome would be appropriate for this client?

Bleeding spontaneously stops within 24 to 48 hours. Explanation: For a threatened miscarriage, an outcome for care would be that all bleeding would spontaneously stop within 24 to 48 hours. Bed rest is not recommended for a threatened miscarriage because blood will pool in the vagina. Vaginal bleeding that saturates a perineal pad in 1 hour is an emergency and could indicate an incomplete or complete miscarriage.

The nurse is orientating in the Labor and Delivery unit and asks her preceptor how to differentiate a client with preeclampsia from one with eclampsia. Which symptoms would the preceptor describe to the new nurse as indicative of severe preeclampsia? Select all that apply.

Blood pressure above 160/110 mm Hg Nondependent edema Hyperactive deep tendon reflexes Explanation: Preeclampsia occurs when a pregnant woman develops hypertension occurring after 20 weeks' gestation and only resolves after the fetus is delivered. Preeclampsia is exhibited by 2+ or more proteinuria, nondependent edema, blood pressure greater than 140 mm Hg systolic and above 90 mm Hg diastolic, and CNS irritability demonstrated by hyperactive deep tendon reflexes. If the client has a seizure, she has moved to eclampsia. Glycosuria is not associated with preeclampsia.

An expectant father confides in the nurse that his pregnant wife, at 10 weeks of gestation, is driving him crazy. "One minute she seems happy, and the next minute she is crying over nothing at all. Is there something wrong with her?" The nurse's best response is: A. "This is normal behavior and should begin to subside by the second trimester." B. "She may be having difficulty adjusting to pregnancy; I will refer her to a counselor I know." C. "This is called emotional lability and is related to hormone changes and anxiety during pregnancy. The mood swings will eventually subside as she adjusts to being pregnant." D. "You seem impatient with her. Perhaps this is precipitating her behavior." c

C. "This is called emotional lability and is related to hormone changes and anxiety during pregnancy. The mood swings will eventually subside as she adjusts to being pregnant."

A woman is 6 weeks pregnant. She has had a previous spontaneous abortion at 14 weeks of gestation and a pregnancy that ended at 38 weeks with the birth of a stillborn girl. What is her gravidity and parity according to the GTPAL system? A. 2-0-0-1-1 B. 2-1-0-1-0 C. 3-1-0-1-0 D. 3-0-1-1-0 c

C. 3-1-0-1-0

Which hematocrit (HCT) and hemoglobin (HGB) results represent the lowest acceptable values for a woman in the third trimester of pregnancy? A. 38% HCT; 14 g/dL HGB B. 35% HCT; 13 g/dL HGB C. 33% HCT; 11 g/dL HGB D. 32% HCT; 10.5 g/dL HGB c

C. 33% HCT; 11 g/dL HGB

In order to reassure and educate pregnant clients about changes in their blood pressure, maternity nurses should be aware that: A. A blood pressure cuff that is too small produces a reading that is too low; a cuff that is too large produces a reading that is too high. B. Shifting the client's position and changing from arm to arm for different measurements produces the most accurate composite blood pressure reading at each visit. C. The systolic blood pressure increases slightly as pregnancy advances; the diastolic pressure remains constant. D. Compression of the iliac veins and inferior vena cava by the uterus contributes to hemorrhoids in the later stage of term pregnancy.

C. The systolic blood pressure increases slightly as pregnancy advances; the diastolic pressure remains constant.

After conducting a refresher class on possible congenital infections with a group of perinatal nurses, the nurse recognizes the class was successful when the group identifies which congenital viral infection as the most common?

CMV

A client at 28 weeks' gestation is seen during a prenatal visit. The nurse notes that the client's most recent hemoglobin level is 10.9 g/dl (109 g/l). Which is an appropriate action for the nurse to take?

Continue with routine prenatal assessments. Explanation: The nurse should continue with routine prenatal assessments for the visit. A hemoglobin level of 10.9 g/dl (109 g/l) during the third trimester is considered a normal finding due to the increased plasma volume that results in a physiologic anemia during pregnancy. This is considered a normal finding, so there is no need to refer the client for additional laboratory testing, notify the health care provider nor instruct the client to increase iron intake.

When counseling a client about getting enough iron in her diet, the maternity nurse should tell her that: A. Milk, coffee, and tea aid iron absorption if consumed at the same time as iron. B. Iron absorption is inhibited by a diet rich in vitamin C. C. Iron supplements are permissible for children in small doses. D. Constipation is common with iron supplements. d

D. Constipation is common with iron supplements.

Which suggestion about weight gain is not an accurate recommendation? A. Underweight women should gain 12.5 to 18 kg. B. Obese women should gain at least 7 kg. C. Adolescents are encouraged to strive for weight gains at the upper end of the recommended scale. D. In twin gestations, the weight gain recommended for a single fetus pregnancy should simply be doubled. d

D. In twin gestations, the weight gain recommended for a single fetus pregnancy should simply be doubled.

A 22-year-old woman pregnant with a single fetus had a preconception body mass index (BMI) of 24. When she was seen in the clinic at 14 weeks of gestation, she had gained 1.8 kg (4 lbs) since conception. How would the nurse interpret this finding? A. This weight gain indicates possible gestational hypertension. B. This weight gain indicates that the woman's infant is at risk for intrauterine growth restriction (IUGR). C. This weight gain cannot be evaluated until the woman has been observed for several more weeks. D. The woman's weight gain is appropriate for this stage of pregnancy.

D. The woman's weight gain is appropriate for this stage of pregnancy.

The nurse is explaining the menstrual cycle to a group of older adolescents. Which factor should the nurse point out will lead to menstruation?

Decreasing production of progesterone by the corpus luteum Explanation: If the ovum is not fertilized, the production of progesterone and estrogen by the corpus luteum begins to decrease. Without the high levels of progesterone and estrogen, the endometrium degenerates and sheds, a process referred to as menstruation. The follicle becomes the corpus luteum after ovulation and continues to secrete estrogen and progesterone. The production of another ovum is not the factor that causes endometrium degeneration. Production of follicle stimulating hormone by the anterior pituitary initiates ovulation.

Which interventions would a pregnant client be taught regarding dietary restrictions during pregnancy? Select all that apply.

Discard foods that have been left out at room temperature for more than 2 hours. Wash raw fruits and vegetables with hot water and a mild soap. Do not drink raw or unpasteurized milk. explanation; While pregnant, women must adhere to certain rules regarding food preparation and storage. The woman should discard foods that have been left out at room temperature for more than 2 hours, wash raw fruits and vegetables with hot water and a mild soap, and avoid drinking raw or unpasteurized milk. Soft cheeses should be avoided. There is no need to limit beef during pregnancy.

The nurse discovers a soft systolic murmur when auscultating the heart of a client at 32 weeks' gestation. Which action would be most appropriate?

Document this and continue to monitor the murmur at future visits. Explanation; Due to the increased blood volume that occurs with pregnancy, soft systolic murmurs may be heard and are considered normal.

A pregnant woman undergoes maternal serum alpha-fetoprotein (MSAFP) testing at 16 to 18 weeks' gestation. What would the nurse suspect if the woman's level is decreased?

Down syndrome Explanation: Decreased levels might indicate Down syndrome or trisomy 18. Sickle cell anemia may be identified by chorionic villus sampling. MSAFP levels would be increased with cardiac defects, such as tetralogy of Fallot. A triple marker test would be used to determine an open neural tube defect.

A pregnant client at 34 weeks' gestation reports a burning sensation in the lower esophagus. What action would the nurse recommend to increase her comfort? Select all that apply.

Eat five to six small meals per day. Do not eat fried, fatty foods. Do not lie down immediately after eating. Explanation: The client is experiencing pyrosis. Eating small frequent meals, avoiding fried foods, and not laying down immediately after eating will minimize the discomfort. Large quantities of carbohydrates and not taking liquids with meals will not change the discomfort being experienced.

A nurse is providing care to a client during the initial prenatal visit. The client has been newly diagnosed with depression and is to begin a regimen of antidepressants and psychotherapy. Which intervention(s) will the nurse implement to further ensure an optimum outcome for this client? Select all that apply.

Encourage the client to keep all prenatal appointments. Provide education on the importance of good nutrition. Reassure the client that depression is a common occurrence in pregnancy. Explanation: To ensure an optimum outcome for the client, the nurse will encourage the client to keep all prenatal appointments and maintain good nutrition. The nurse will also reassure the client that depression is very common even in pregnancy and is nothing to be ashamed of. Antidepressants are not a contraindication to breastfeeding. While monoamine oxidase inhibitors (MAOIs) are antidepressants, they are not used in pregnancy because of their teratogenic effects on the fetus.

A woman in labor has brought a butcher knife with her and insists on putting it under her hospital mattress to "cut the labor pain." Which is the nurse's best response to this situation?

Explain the security issue and negotiate with the client asking if a picture or rubber/play knife would honor her values and beliefs. Explanation: This situation highlights the need for nurses to be aware of culturally held beliefs and practices of the clients they see and provide care for. Respect for cultural values is important for developing effective nurse-client relationships. The nurse should allow the client to believe what she wishes is appropriate. An attempt to educate the client or outright restriction of her desires would worsen the situation.

A nurse is assessing a pregnant client. The nurse understands that hormonal changes occur during pregnancy. Which hormones would the nurse most likely identify as being inhibited during the pregnancy?

FSH and LH

The nurse is teaching a group of women about the hormone that makes the ovum within the ovaries mature each month. Which hormone is responsible for stimulating the ovum?

Follicle-stimulating hormone

During what phase of the uterine cycle would the uterine lining begin to have a reduced blood supply?

Ischemic phase On Day 27 and 28 of the uterine cycle, the corpus luteum is no longer producing either estrogen or progesterone. Without these hormones to maintain the blood vessels in the uterus, the uterine lining begins to become ischemic or without blood supply, initiating the ischemic phase.

The nursing student correctly identifies what information about oogenesis?

It begins in the ovaries before birth but is not fully complete until the childbearing years.

A woman who is 10 weeks' pregnant calls the physician's office reporting "morning sickness" but, when asked about it, tells the nurse that she is nauseated and vomiting all the time and has lost 5 pounds. What interventions would the nurse anticipate for this client?

Lab work will be drawn to rule out acid-base imbalances.

A client who is at 37 weeks' gestation is directed to perform a daily fetal movement (kick) count. What nursing instruction will be included in the teaching plan? Select all that apply.

Lie down to do the count after eating a meal. Count fetal movements until a total of 10 are counted and record the length of time. Explanation:A healthy fetus moves about 10 times per hour. The nurse should instruct the client to lie in a left recumbent position after a meal, observe and record the number of fetal movements or kicks the fetus makes until 10 movements are counted, and record the length time. If an hour passes without 10 movements, the client should walk around a little and try a count again. If 10 movements cannot be felt in a second 1-hour period, the client should telephone the primary health care provider.

Which statement should be incorporated into the teaching plan developed to present instruction about the female menstrual cycle to nursing students?

Menses ensues when the levels of estrogen and progesterone fall.

A client at 25 weeks' gestation presents with a blood pressure of 152/99 mm Hg, pulse 78 beats/min, no edema, and urine negative for protein. What would the nurse do next?

Notify the health care provider

The nurse understands the need to be aware of the potential of bleeding disorders in pregnant clients. Which disorder should she be aware of that occurs in the second trimester?

Placenta previa Second trimester bleeding usually results from placenta previa, where the placenta lies either partially or completely over the cervical os. The pregnant client begins to experience vaginal bleeding of bright, red blood. Spontaneous abortion (miscarriage), hydatidiform mole, and ectopic pregnancy occur in the first trimester and cervical insufficiency is not a bleeding disorder.

After a regular prenatal visit, a pregnant client asks the nurse to describe the differences between placental abruption (abruptio placentae) and placenta previa. Which statement will the nurse include in the teaching?

Placenta previa is an abnormally implanted placenta that is too close to the cervix. Placenta previa is a condition of pregnancy in which the placenta is implanted abnormally in the lower part of the uterus and is the most common cause of painless, bright red bleeding in the third trimester. Placental abruption is the premature separation of a normally implanted placenta that pulls away from the wall of the uterus either during pregnancy or before the end of labor. Placental abruption can result in concealed or apparent dark red bleeding and is painful. Immediate intervention is required for placental abruption.

A nurse is providing care to a client diagnosed with a Class III cardiac disability who is in the second stage of labor. Which intervention will the nurse take?

Position the client for passive descent.

A 17-year-old primigravida with type 1 diabetes is at 37 weeks' gestation comes to the clinic for an evaluation. The nurse notes her blood sugar has been poorly controlled and the health care provider is suspecting the fetus has macrosomia. The nurse predicts which step will be completed next?

Preparing for amniocentesis and fetal lung maturity assessment

Group B streptococcus (GBS) infection presents a large risk to the neonate. Which factor should the nurse consider when developing a plan of care related to GBS? Select all that apply.

Preterm labor clients receive prophylactic antibiotics. Antibiotics must be started 4 hours prior to labor to be effective. Mothers with previous GBS will be treated with prophylactic antibiotics.

A female client states, "I am concerned because when I have sex with my spouse, I have an area near my vagina that is really sensitive to touch." Which action will the nurse complete?

Provide education to the client on the female anatomy.

A woman calls the obstetrician's office and states that she is feeling nauseated and tired and suspects she may be pregnant. The nurse asks the woman when her last menstrual cycle was and is told that the woman had a light cycle three weeks ago. How would the nurse respond to this client?

Recommend that the woman come into the office for a pregnancy test or do a home pregnancy test.

The nurse is caring for an intrapartum mother whose fetus has asymmetrical intrauterine growth restriction (IUGR) after the 24th week of gestation. Which nursing action is best?

Regular assessment of the fetal monitor tracings and preparation for a cesarean birth, if needed.

A nurse is describing the use of Rho(D) immune globulin as the therapy of choice for isoimmunization in Rh-negative women and for other conditions to a group of nurses working at the women's health clinic. The nurse determines that additional teaching is needed when the group identifies which situation as an indication for Rho(D) immune globulin?

STIs Explanation: Indications for Rho(D) immune globulin include isoimmunization, ectopic pregnancy, chorionic villus sampling, amniocentesis, prenatal hemorrhage, molar pregnancy, maternal trauma, percutaneous umbilical sampling, therapeutic or spontaneous abortion, fetal death, or fetal surgery.

The nurse is documenting subjective and objective data changes from a client at 34 weeks' gestation. Which would the nurse report immediately to the health care provider? Select all that apply.

Scant spotting on underwear Sharp abdominal pain explanation; The nurse is correct to report to the health care provider any signs of vaginal bleeding and sharp abdominal pain as these could indicate an emergency. Normal common discomforts of pregnancy include difficulty sleeping due to the increased size of the abdomen, feeling faint due to postural hypotension, bleeding gums and nosebleeds due to hormonal and drying effects of pregnancy. While it is common to have sharp round ligament pain from the increasing pregnancy, it is usually on the right side and must be further examined.

A woman who has sickle cell anemia asks the nurse if her infant will develop sickle cell disease. The nurse would base the answer on which information?

Sickle cell anemia is recessively inherited. Explanation: Sickle cell anemia is an autosomal recessive disease requiring that the person have two genes for the disease, one from each parent. If one parent has the disease and the other is free of the disease and trait, the chance of the child inheriting the disease is zero. If the woman has the disease and her partner has the trait, there is a 50% chance that the child will be born with the disease. If both parents have the disease, then all of their children also will have the disease.

A pregnant woman at 12 weeks' gestation comes to the office reporting she has begun minimal fresh vaginal spotting. She is distressed because her primary care provider indicates after examining her that they will "wait and see." Which response would be most appropriate from the nurse in answering this client's concerns?

Tell her that medication to prolong a 12-week pregnancy usually is not advised. Explanation: Because many early pregnancy losses occur as the result of chromosome abnormalities, an aggressive approach to prolong these is not usually recommended. It would not be appropriate for the nurse to suggest an over-the-counter tocolytic, nor to tell the client that the care provider meant something else such as maintaining strict bed rest. Advising the client to seek a second opinion would not change the end results.

A client at 29 weeks' gestation tells the nurse she is experiencing aches in her hips and joints. What would the nurse do next?

Tell the client these are normal findings during pregnancy.

The nurse provides instructions to a client with hyperemesis gravidarum. Which outcome indicates that teaching has been effective?

The client is able to ingest a regular diet after progressing through clear liquids and soft foods.

A physician has prescribed magnesium sulfate for a client with premature labor. Data collection reveals the client's respiratory rate is 12 breaths/minute, and urine output is 30 ml/hour. The magnesium sulfate serum levels are 7 mg/dl. When questioned, the client reports feeling warm and flushed. Based upon the nurse's understanding of magnesium sulfate, what action is most appropriate?

The client's response is appropriate and within normal limits; therefore, no action is necessary. Explanation: Magnesium sulfate is associated with feelings of warmth and flushing; these symptoms do not indicate toxicity or allergic reaction. Respirations of 12 breaths/minute are considered normal. Urine output should be at least 30 ml/hour. Serum levels of magnesium sulfate should be between 4 and 8 mg/dl to promote a therapeutic response.

A physician has prescribed magnesium sulfate for a client with premature labor. Data collection reveals the client's respiratory rate is 12 breaths/minute, and urine output is 30 ml/hour. The magnesium sulfate serum levels are 7 mg/dl. When questioned, the client reports feeling warm and flushed. Based upon the nurse's understanding of magnesium sulfate, what action is most appropriate?

The client's response is appropriate and within normal limits; therefore, no action is necessary. Explanation:Magnesium sulfate is associated with feelings of warmth and flushing; these symptoms do not indicate toxicity or allergic reaction. Respirations of 12 breaths/minute are considered normal. Urine output should be at least 30 ml/hour. Serum levels of magnesium sulfate should be between 4 and 8 mg/dl to promote a therapeutic response.

The nurse is assessing the past history of a woman at her first prenatal appointment. If the woman has no history of vaccination of the choices listed, which vaccine would the nurse schedule this client to receive?

The flu vaccine, after the first trimester of pregnancy is complete

A woman is going through the luteal phase of her ovarian cycle. What changes are taking place during this period of time?

The follicle develops into the corpus luteum. In the luteinizing phase, ovulation has already occurred and if the egg is fertilized, progesterone and estrogen levels increase. The follicle remnants become the corpus luteum, a yellow body on the ovary. LH levels remain elevated after ovulation. FSH levels increase during the follicular stage.

The nurse is transcribing messages from the answering service. Which phone message should the nurse return first?

a 35-year-old, 21-week G3P2 client with blood pressure of 160/110 mm Hg, blurred vision, and whose last blood pressure was 143/99 mm Hg and urine dipstick showed a +2 proteinuria

A pregnant client at 34 weeks' gestation has systemic lupus erythematosus (SLE). The client has undergone a nonstress test (NST) for fetal well-being. The nurse is reviewing the test results, which show a single fetal heart rate acceleration of 15 beats lasting 15 seconds within a 20-minute period. How will the nurse interpret these results?

The test is nonreactive and should continue for at least 40 minutes.

A pregnant client at 34 weeks' gestation has systemic lupus erythematosus (SLE). The client has undergone a nonstress test (NST) for fetal well-being. The nurse is reviewing the test results, which show a single fetal heart rate acceleration of 15 beats lasting 15 seconds within a 20-minute period. How will the nurse interpret these results?

The test is nonreactive and should continue for at least 40 minutes. Explanation: The results of the nonstress test (NST) are nonreactive and should continue for at least 40 minutes in hopes of eliciting two fetal heart accelerations of 15 beats lasting 15 seconds each. The test would then be considered reactive. Further evaluation would likely include a biophysical profile and additional NSTs during the final 4 to 6 weeks of pregnancy. NST results are either reactive or nonreactive; contraction stress test (CST) results can be positive, negative, or equivocal (or indeterminate) and require further evaluation with a biophysical profile.

A pregnant client at 34 weeks' gestation has systemic lupus erythematosus (SLE). The client has undergone a nonstress test (NST) for fetal well-being. The nurse is reviewing the test results, which show a single fetal heart rate acceleration of 15 beats lasting 15 seconds within a 20-minute period. How will the nurse interpret these results?

The test is nonreactive and should continue for at least 40 minutes. Explanation:The results of the nonstress test (NST) are nonreactive and should continue for at least 40 minutes in hopes of eliciting two fetal heart accelerations of 15 beats lasting 15 seconds each. The test would then be considered reactive. Further evaluation would likely include a biophysical profile and additional NSTs during the final 4 to 6 weeks of pregnancy. NST results are either reactive or nonreactive; contraction stress test (CST) results can be positive, negative, or equivocal (or indeterminate) and require further evaluation with a biophysical profile.

A young woman presents at the emergency department reporting lower abdominal cramping and spotting at 12 weeks' gestation. The primary care provider performs a pelvic examination and finds that the cervix is closed. What does the care provider suspect is the cause of the cramps and spotting?

Threatened abortion

The nurse is reviewing the process of oocyte maturation and ovulation with a client. What occurs during the follicular phase of the ovarian cycle that the nurse should include in the teaching session?

Under the influence of follicle-stimulating hormone, several follicles begin to ripen, and the ovum with each begins to mature. The follicular phase lasts from about day 4 to about day 14. During this time, under the influence of follicle-stimulating hormone, several follicles begin to ripen and the ovum within each begins to mature. About day 14, a surge of hormones causes the ovum to burst through the ovary; this act is called ovulation. During the luteal phase, the empty, ruptured graafian follicle becomes the corpus luteum, and it begins to secrete progesterone and estrogen. The endometrium of the uterus has a similar cycle. It is called the uterine cycle or endometrial cycle. This process prepares the uterus for implantation of an ovum (egg).

A high-risk 43-year-old client is scheduled to receive amnioinfusion due to oligohydramnios. Which prescription by the provider should the nurse question before implementing it?

Use clean technique when handling the double-lumen catheter and IV tubing.

A woman in her first trimester of pregnancy has noted an increase in a thick, whitish vaginal discharge even though she showers daily. The woman shares this information with the clinic nurse, who provides some client education on the topic of leukorrhea. Which interventions should be addressed in this discussion? Select all that apply.

Wear cotton underwear during the day. A perineal pad to absorb the discharge may help. Explanation; Leukorrhea, a whitish, viscous vaginal discharge or an increase in the amount of normal vaginal secretions, occurs in response to the high estrogen level and the increased blood supply to the vaginal epithelium and cervix in pregnancy. Wearing cotton underpants and sleeping at night without underwear can be helpful. Some women may need to wear a perineal pad to absorb the discharge. Caution women not to use tampons because this could lead to stasis of secretions and subsequent infection. Advise women to contact their obstetric provider if there is a change in the color, odor, or character of this discharge as these suggest infection. Caution women not to douche; douching is contraindicated generally, and especially throughout pregnancy.

A school nurse is teaching a sex education class for middle school children. When providing education about menstruation, what information should the nurse include? Select all that apply.

Women should calculate ovulation on the 14th day of their cycle. The hypothalamus, pituitary gland, ovaries, and uterus control menstruation.

The nurse is caring for a client at 30 weeks' gestation who has a history of recurrent cardiac failure and an average blood pressure of 96/58 mm Hg. The primary health care provider is concerned about the status of the mother and neonate. Which neonatal objective data is the nurse correct to identify as related to inadequate supply of blood to the placenta?

a current fundal height of 22 cm Explanation: Cardiac failure can affect fetal growth at the point at which maternal blood pressure becomes insufficient to provide an adequate supply of blood and nutrients to the placenta. For this reason, the infant may tend to have a low fundal height, indicating a low birth weight. Feeling 12 kicks per hour and a heart rate of 145 beats per minute are normal findings. The client should feel fetal movement at least hourly.

Which of the following changes, with highest priority, should the nurse teach a pregnant client to report to the health care provider as soon as possible?

abdominal pain coming and going during the third trimester

What is the best indicator that the client is experiencing an ectopic pregnancy?

adnexal tenderness

A woman's obstetrician prescribes vitamin K supplements for a client who is on antiepileptic medications beginning at 36 weeks' gestation. The mother asks the nurse why she is taking this medication. The nurse's best response would be:

antiepileptic therapy can lead to vitamin K-deficient hemorrhage of the newborn.

A 19-year-old woman presents to the emergency department in the late stages of active labor. Assessment reveals she received no prenatal care. As part of her examination, a rapid HIV screen indicates she is HIV positive. To reduce the perinatal transmission to her infant, which intravenous medication would the nurse anticipate administering?

antiretroviral

A client at 28 weeks' gestation is admitted for an exacerbation of asthma. Which test(s) will the nurse monitor to determine the severity of the exacerbation? Select all that apply.

arterial blood gases (ABGs) pulse oximetry peak flow measurements Explanation: A client who is admitted with an exacerbation of asthma during pregnancy will have continuous pulse oximetry with a goal of maintaining an oxygen saturation level of 95%. When evaluating the blood gases during an asthma exacerbation in pregnancy, it is important to remember that some respiratory alkalosis is normal in pregnancy. Peak flow is also used to objectively assess the severity of the exacerbation. Hemoglobin and serum carbon dioxide levels are not used to determine the severity of asthma exacerbation in a pregnant client.

A client with systemic lupus erythematosus (SLE) is receiving postpartum care after the birth process. Which medication does the nurse anticipate being prescribed for this client?

aspirin Explanation: Because of the high risk for preeclampsia in a client with systemic lupus erythematosus (SLE), low-dose aspirin therapy (81 mg daily) is recommended starting between 12 and 20 weeks' postpartum. Diuretics and beta blockers are not routinely prescribed for pregnant clients with SLE. Glucocorticoids would be used to treat fetal heart block.

When developing a plan of care, the nurse needs to make which assessments for a pregnant client with a BMI of 18.3? Select all that apply.

assess for maternal fatigue assess for poor fetal growth assess for preterm labor

A potential complication for the mother and fetus is Rh incompatibility; therefore, assessment should include blood typing. If the mother is Rh negative, her antibody titer should be evaluated. If treatment with Rho(D) immune globulin is indicated, the nurse would expect to administer it at which time?

at 28 weeks

A nurse is teaching a 30-year-old gravida 1 who has sickle cell anemia. Providing education on which topic is the highest nursing priority?

avoidance of infection

A gravida 2 para 1 client with a history of gestational hypertension chooses to not add any salt to her current diet, even though her blood pressure has been within normal limits. The nurse supports the client's decision but instructs the client to consume what foods?

cranberries and seafood Explanation: Iodine is added to table salt and is needed for thyroid function. Cranberries and seafood contain iodine.

Human papillomavirus (HPV) can cause condylomata acuminata that can develop in clusters on the vulva, within the vagina, on the cervix, or around the anus. What is their risk?

block a vaginal birth

A high-risk pregnant client is determined to have gestational hypertension. The nurse suspects that the client has developed preeclampsia with severe features based on which finding?

blurred vision Explanation: Visual symptoms such as blurred vision and blind spots suggest severe preeclampsia. Severe preeclampsia is characterized by a blood pressure of 160/110 mm Hg. Mild facial edema or hand edema occurs with mild preeclampsia. Proteinuria in severe preeclampsia is greater than 500 mg per 24 hours.

The nurse is caring for a postpartum client with uterine atony. Bladder drainage and massage have been ineffective. Oxytocin IV has been given but has been ineffective in maintaining uterine tone. Which medication does the nurse anticipate being prescribed as the next choice?

carboprost tromethamine Explanation; If oxytocin is not effective at maintaining tone, carboprost tromethamine, a prostaglandin F2-alpha derivative, or methylergonovine maleate, an ergot compound, both given intramuscularly, are the next possible options. Additional options include misoprostol, a prostaglandin E1 analogue, administered rectally to decrease postpartum hemorrhage or ranexamic acid to decrease bleeding. Heparin would increase bleeding and would not be used.

In preparing a class for a group of pregnant couples, the nurse includes information about possible newborn complications associated with smoking during pregnancy. Which complications will the nurse include? Select all that apply.

cerebral palsy low birth weight cleft lip and palate sudden infant death syndrome Explanation:Smoking during pregnancy is associated with multiple perinatal and childhood risks, including low birth weight, SIDS, cerebral palsy, cleft lip and palate, clubfoot (congenital talipes equinovarus), asthma, altered brainstem development, middle ear infections, and reduced head circumference. Trisomy 21, or Down syndrome, is a genetic disorder caused by abnormal cell division, not the effects of nicotine.

The nurse is reviewing the lab results for a client at 6 weeks' gestation based on the date of her last menstrual period and a basal body temperature chart. The client's HcG level is 160,000 mlU/ml (160,000 IU/L). Based on this result, the nurse anticipates the client will have an obstetrical ultrasound examination to determine if the client has which condition?

choriocarcinoma Explanation: HcG is considered a marker, which means it is a substance that is excreted by another tissue. In early pregnancy, HcG is produced by the developing embryo. At 6 weeks' gestation, the expected range is 1,080 to 56,500 mIU/ml (1,080 to 56,500 IU/L). An elevated value is associated with choriocarcinoma or multiple-gestation pregnancy. A low level is associated with ectopic pregnancy or a blighted ovum. Granulosa cell tumor is an ovarian tumor and does not increase HcG levels.

A pregnant client with a history of heart disease has been admitted to a health care center reporting breathlessness. The client also reports shortness of breath and easy fatigue when doing ordinary activity. The client's condition is markedly compromised. The nurse would document the client's condition using the New York Heart Association (NYHA) classification system as which class?

class III

A young client with heart disease asks the physician if it would be safe for her to have a baby. The physician responds that it is possible but she would have to maintain almost complete bed rest. Which classification of heart disease is this woman?

class III

A woman with cardiac disease has come to the office for prenatal counseling. Assessment supports the decision to caution the woman against pregnancy. The woman most likely fits the criteria for which functional risk classification?

class IV

A client at 20 weeks' gestation experiences a miscarriage. For the use of which substance should the nurse assess this client?

cocaine Explanation: Miscarriage is an obstetrical risk for the use of cocaine. Alcohol can cause low birth weight, small for gestational age, preterm birth, and stillbirth. Caffeine is not associated with any particular obstetrical risk. Vitamin supplements are not identified as causing any specific obstetrical risk.

A pregnant client with multiple gestation arrives at the maternity clinic for a regular antenatal check up. The nurse would be aware that client is at risk for which perinatal complication?

congenital anomalies Explanation: Multiple gestation involves two or more fetuses. The perinatal complications associated with multiple pregnancy include preterm birth, maternal hypertension and congenital anomalies. Fetal nonimmune hydrops occurs in the infection of pregnant clients with parvovirus. Postterm birth, maternal hypotension, and fetal nonimmune hydrops are not seen as complications of multiple pregnancy.

A woman is admitted to the labor suite with contractions every 5 minutes lasting 1 minute. She is postterm and has oligohydramnios. What does this increase the risk of during birth?

cord compression Oligohydramnios and meconium staining of the amniotic fluid are common complications of postterm pregnancy. Oligohydramnios increases the incidence of cord compression, which can lead to fetal distress during labor.

A nurse assesses a primigravida client in the eighth week of gestation. The client reports nausea and vomiting in the mornings. The client tells the nurse, "I'm not able to keep liquids down and I'm eating like a bird." The client also expresses concerns about hormonal changes and how the pregnancy will affect her physical appearance. Which client problem should the nurse assess first?

deficient fluid volume

A nurse is assessing a pregnant client for the possibility of preexisting conditions that could lead to complications during pregnancy. The nurse suspects that the woman is at risk for hydramnios based on which preexisting condition?

diabetes Explanation: Approximately 18% of all women with diabetes will develop hydramnios during their pregnancy. Hydramnios occurs in approximately 2% of all pregnancies and is associated with fetal anomalies of development.

A nurse is preparing a postpartum client for discharge who was diagnosed with new-onset peripartum cardiomyopathy. Related to the client's diagnosis, what information will the nurse include in the discharge instructions? Select all that apply.

directions on how to take prescribed vasodilators education on signs/symptoms of worsening condition information regarding dietary restrictions Explanation: The nurse will include information on dietary sodium and fluid restrictions, directions regarding how to take prescribed vasodilators, and signs/symptoms of a worsening condition. A cardiac surgical repair would be needed for structural defects; cardiomyopathy is not a structural problem. Newborn congenital anomalies are not associated with peripartum cardiomyopathy.

The nurse is preparing to assess a young woman's external genitalia. The nurse predicts the client is currently in the most fertile phase of the menstrual cycle based on which assessment finding of the vaginal mucus drainage?

distensible, stretchable quality

A client at 32 weeks' gestation receives an ultrasound that identifies intrauterine growth restriction. Which findings from the client's nutritional assessment would indicate to the nurse that additional teaching is needed? Select all that apply.

eating large quantities of empty-calorie foods difficulty eating because of continuing nausea maternal age less than 18 years

A client at 32 weeks' gestation receives an ultrasound that identifies intrauterine growth restriction. Which findings from the client's nutritional assessment would indicate to the nurse that additional teaching is needed? Select all that apply.

eating large quantities of empty-calorie foods difficulty eating because of continuing nausea maternal age less than 18 years Explanation:Low caloric intake because of continued nausea as well as eating large quantities of empty calories can impact fetal growth. Pregnant adolescents also are more likely to have poor fetal growth because the adolescent's growth needs are competing with the fetus for nutrients. Consuming small, frequent meals is a strategy for increasing caloric and nutrient intake during pregnancy. A history of gestational diabetes makes it more likely the client will experience elevated blood glucose levels and fetal macrosomia.

A nurse is teaching a group of pregnant woman about bleeding that can occur early in pregnancy. The nurse determines that additional teaching is needed when the group identifies which condition as a common cause?

placenta previa Explanation: The three most common causes of hemorrhage during the first half of pregnancy are spontaneous abortion (miscarriage), ectopic pregnancy, and GTD. Placenta previa occurs in the later weeks of gestation.

The nursing student is preparing a presentation that will illustrate the various stages of fetal development. The student will label which stage as the time when the various tissues of the growing embryo begin to assume specific functions?

embryonic Explanation: During the embryonic period, the cells multiply and tissues begin to assume specific functions, a process known as differentiation. This stage lasts from the end of the second week after fertilization until the end of the eighth week. The pre-embryonic stage begins with fertilization and lasts for 2 weeks. Cellular division and implantation occur during this stage of development. The fetal stage is from the beginning of the ninth week after fertilization and continues until birth. There is additional growth and maturation of the organs and body systems during this time. There is no post-embryonic stage.

The nursing instructor is teaching a session comparing the various phases of the menstrual and uterine cycles. The instructor determines the session is successful when the students correctly choose which hormone that is responsible for regulating the proliferative phase of the uterine cycle?

estrogen Explanation: Estrogen is the hormone that influences the proliferative phase. During the secretory phase, the corpus luteum produces progesterone. Follicle-stimulating hormone controls the follicular phase and luteinizing hormone controls the luteal phase of the ovarian cycle.

Which finding is most worrisome in a client in her 26th week of pregnancy?

facial edema Explanation: Generalized hair loss, hyperpigmented maxillary rash (chloasma), and nosebleeds are usually benign and common in pregnancy. Facial edema after the 24th week of gestation may indicate gestational hypertension.

A client with diabetes is in the third trimester of pregnancy. Which assessment finding indicates that the client may be experiencing vasculopathy?

fetal growth restriction Explanation: In the third trimester, a pregnant client will have more frequent examinations. If the client has poor glycemic control, fetal hypoxia may result from hyperglycemia in the pregnant parent that leads to fetal hyperglycemia and increased insulin production and oxygen consumption, or from a reduction in uteroplacental perfusion resulting from diabetic vasculopathy. Vasculopathy may first be evidenced by fetal growth restriction. Vision changes could indicate the development of diabetic retinopathy. An elevated blood pressure could indicate the development of chronic hypertension. Premature rupture of membranes (PROM) can occur in a variety of conditions and not necessarily in a client with diabetes.

A pregnant client with deep vein thrombosis has been diagnosed as having systemic lupus erythematosus (SLE). The nurse would monitor the client closely for the development of which complication?

fetal malnutrition

A nurse is admitting a pregnant woman with sickle cell anemia to the emergency department. Which findings would lead the nurse to suspect the client is in crisis? Select all that apply.

fever joint pain fatigue Explanation:Signs and symptoms of a sickle cell crisis commonly include severe abdominal pain, muscle spasm, leg pains, joint pain, fever, stiff neck, nausea and vomiting, and seizures. Skin turgor would most likely be poor because the client would probably be dehydrated. The client may also be fatigued during the crisis. Pallor would be the result of the anemia but not necessarily indicative of a crisis.

A pregnant woman is diagnosed with placental abruption (abruptio placentae). When reviewing the woman's physical assessment in her medical record, which finding would the nurse expect?

firm, rigid uterus on palpation Explanation; The uterus is firm-to-rigid to the touch with abruptio placentae. It is soft and relaxed with placenta previa. Bleeding associated with abruptio placentae occurs suddenly and is usually dark in color. Bleeding also may not be visible. A gradual onset of symptoms is associated with placenta previa. Fetal distress or absent fetal heart rate may be noted with abruptio placentae. The woman with abruptio placentae usually experiences constant uterine tenderness on palpation.

A nurse is conducting an in-service education program about the female reproductive cycle for a group of nurses working at a women's health clinic. When describing the phases of the endometrial cycle, the nurse would explain that the proliferative phase of the endometrial cycle corresponds to which phase in the ovarian cycle?

follicular

After teaching a health education class on the female reproductive cycle, the nurse determines that the teaching was effective when the group identifies which phase as belonging to the ovarian cycle?

follicular phase

A woman at 10 weeks' gestation comes to the clinic for an evaluation. Which assessment finding should the nurse prioritize?

fundal height measurement of 18 cm Explanation: A fundal height of 18 cm is larger than expected and should be further investigated for gestational trophoblastic disease (hydatidiform mole). One of the presenting signs is the uterus being larger than expected for date.

A client questions the nurse about how her fetus's gender is determined. The nurse's most accurate reply would be that gender of the fetus is determined when:

gametes from the mother and gametes from the father are joined during meiosis.

Which assessment finding will alert the nurse to be on the lookout for possible placental abruption (abruptio placentae) during labor?

gestational hypertension Risk factors for placental abruption (abruptioo placentae) include preeclampsia, gestational hypertension, seizure activity, uterine rupture, trauma, smoking, cocaine use, coagulation defects, previous history of abruption, intimate partner violence, and placental pathology. Macrosomia, gestational diabetes, and low parity are not considered risk factors.

A pregnant client has been admitted with reports of brownish vaginal bleeding. On examination, there is an elevated human chorionic gonadotropin (hCG) level, absent fetal heart sounds, and a discrepancy between the uterine size and the gestational age. The nurse interprets these findings to suggest which condition?

gestational trophoblastic disease

A client at 24 weeks' gestation is hospitalized for treatment of diabetic ketoacidosis. The client has a glucose of 220 mg/dl (12.21 mmol/l) and is experiencing nausea and vomiting. The electronic fetal monitor shows a fetal heart rate with no accelerations and several late decelerations. What action does the nurse take?

glucose control Explanation:The nurse will prioritize glucose control. Diabetic ketoacidosis can occur with glucose levels as low as 200 mg/dl (11.10 mmol/l) in pregnancy. Once the client's glucose level is under control, the nausea and vomiting should resolve, negating the need for fluid volume replacement. Diabetic ketoacidosis is not an indication for an emergent birth, and any abnormal aspects of the fetal heart tracing usually resolves once the pregnant client's condition improves.

A client has been admitted with placental abruption (abruptio placentae). She has lost 1,200 ml of blood, is normotensive, and ultrasound indicates approximately 30% separation. The nurse documents this as which classification of abruptio placentae?

grade 2 The classifications for placental abruption (abruptio placentae) are: grade 1 (mild) - minimal bleeding (less than 500 ml), 10% to 20% separation, tender uterus, no coagulopathy, signs of shock or fetal distress; grade 2 (moderate) - moderate bleeding (1,000 to 1,500 ml), 20% to 50% separation, continuous abdominal pain, mild shock, normal maternal blood pressure, maternal tachycardia; grade 3 (severe) - absent to moderate bleeding (more than 1,500 ml), more than 50% separation, profound shock, dark vaginal bleeding, agonizing abdominal pain, decreased blood pressure, significant tachycardia, and development of disseminated intravascular coagulopathy. There is no grade 4.

The nurse is preparing the client for the routine laboratory tests that will be obtained at the first prenatal visit. Which test will the nurse prioritize at this visit?

hepatitis screen

Which STI could be transmitted perinatally?

herpes simplex

TORCH is an acronym for maternal infections associated with congenital malformations and disorders. Which disorder does the H represent?

herpes simplex virus

A primigravida at 28 weeks' gestation comes to the clinic for a checkup. She tells the nurse that her mother gave birth to both of her children prematurely, and she is afraid that the same will happen to her. Which risk factors associated with preterm birth would the nurse discuss with the client? Select all that apply.

history of previous preterm birth current multiple gestation pregnancy uterine or cervical abnormalities The top three risk factors for premature birth are history of previous preterm birth, current multiple gestation pregnancy, and uterine or cervical abnormalities.

A client at 34 weeks' gestation is seen in the clinic for a prenatal visit. The nurse is reviewing the client's history and notes the client has sickle cell anemia. Which aspect of the client's care will the nurse prioritize to ensure an optimum pregnancy outcome?

hydration

A nurse is conducting a class on gestational diabetes for a group of pregnant women who are at risk for the condition. The nurse determines that additional teaching is needed when the class identifies which complication as affecting the neonate?

hyperglycemia

A nurse is assessing a pregnant woman with gestational hypertension. Which finding would lead the nurse to suspect that the client has developed severe preeclampsia?

hyperreflexia Explanation: Severe preeclampsia is characterized by blood pressure over 160/110 mm Hg, urine protein levels greater than 500 mg/24 hours, and hyperreflexia. Mild facial edema is associated with mild preeclampsia.

A nursing instructor is conducting a session exploring the signs and symptoms of eclampsia to a group of student nurses. The instructor determines the session is successful after the students correctly choose which signs indicating eclampsia? Select all that apply.

hyperreflexia blurring of vision proteinuria

A nurse is monitoring for signs of cardiac decompensation in a client who is laboring and who has a history of cardiovascular disease. The nurse should be especially vigilant at which point during the labor and birth process?

immediate postpartum Explanation: Immediately postpartum cardiac output increases 65% over the baseline, increasing the risk for cardiac decompensation. Cardiac output increases 15% to 30% during the first stage of labor and 45% during the second stage over the baseline. At 60 minutes postpartum, cardiac output drops to 40% over the baseline.

The nurse is assessing client status and behavior as the client attends monthly visits to the obstetrician. At which time during pregnancy does the nurse anticipate the client changing the verbiage about the fetus, from referring to an object to referring to a human pronoun?

in the second trimester after quickening occurs Explanation: The nurse assesses the pregnant parent not only physically but psychosocially as well. The nurse watches for clues that the client is accepting the child. This most often occurs in the second trimester after quickening occurs. Quickening allows for the client to understand the child's presence and proof of existence. Typical confirmation of pregnancy occurs early with little outward changes occurring to the pregnant parent. Acceptance should occur prior to the imminent birth of the fetus, when the fetus is engaged and labor is anticipated. The family is told at varying times. Nesting occurs in the third trimester when preparation for the newborn occurs.

A nurse conducting a presentation for a group of nurses explains the changes in various body systems related to pregnancy. The nurse determines that additional teaching is needed when the group chooses which component as contributing to the pregnant woman's hypercoagulable state?

increased number of red blood cells explanation; The increase in red blood cells is necessary to transport the additional oxygen required during pregnancy. It has no effect on the hypercoagulable state. Both fibrin and plasma fibrinogen levels increase, along with various blood clotting factors, leading to a hypercoagulable state.

The nursing instructor is presenting the basic physiologic changes in the woman that can occur during a pregnancy. The instructor determines the session is successful when the students correctly choose which change in the respiratory function during pregnancy as normal?

increased tidal volume

Preterm prelabor rupture of membranes (PPROM) can be a serious complication of labor. What is the most common cause of preterm PROM?

infection

During a routine antepartal visit, a pregnant woman says, "I've noticed my gums bleeding a bit since I've become pregnant. Is this normal?" The nurse bases the response on the understanding of which effect of pregnancy?

influence of estrogen and blood vessel proliferation explanation; During pregnancy, the gums become hyperemic, swollen, and friable and tend to bleed easily. This change is influenced by estrogen and increased proliferation of blood vessels and circulation to the mouth. Elevated progesterone levels cause smooth muscle relaxation, which results in delayed gastric emptying and decreased peristalsis. Increased venous pressure contributes to the formation of hemorrhoids. Relaxation of the cardiac sphincter, in conjunction with slowed gastric emptying, leads to reflux due to regurgitation of the stomach contents into the upper esophagus.

What abnormal laboratory values would a nurse expect to find in a woman with hyperemesis gravidarum?

ketonuria Explanation; Women with hyperemesis gravidarum may have an increased hematocrit owing to hemoconcentration from being severely dehydrated. Severe vomiting may lead to hypokalemic acidosis, which can produce a decrease in sodium, potassium, or chloride. A low level of sodium, potassium, or chloride may also result from decreased oral intake.

The nurse reviews the medication therapy regimen of a pregnant woman with chronic hypertension. Which medication would the nurse most likely expect to find?

labetalol

Which measure would the nurse include in the plan of care for a woman with prelabor rupture of membranes if her fetus's lungs are mature?

labor induction

When teaching a class of pregnant women about the effects of substance use during pregnancy, the nurse would include which effect?

low-birth-weight infants Explanation: Substance use during pregnancy is associated with low-birth-weight infants, preterm labor, spontaneous abortion (miscarriage), intrauterine growth restriction, placental abruption (abruptio placentae), neurobehavioral abnormalities, and long-term childhood developmental consequences. Excessive weight gain, higher pain tolerance, and longer gestational periods are not associated with substance use.

During a routine prenatal checkup, the nurse interviews a pregnant client to identify possible risk factors for developing gestational diabetes. Which factor(s) will the nurse identify as increasing the woman's risk? Select all that apply.

maternal obesity with body mass index more than 35 African heritage previous history of spontaneous abortion (miscarriage) Explanation; The risk factors for gestational diabetes include previous history of spontaneous abortion (miscarriage), maternal obesity with body mass index (BMI) more than 35, and client of a high-risk ethnic group such as Black, Native American/First Nations, Latino, and Asian. The other risk factors for gestational diabetes are previous history of stillbirth, birth of large-for-gestational-age neonate, and advancing maternal age.

Which vaccines are contraindicated during pregnancy since they may transmit a viral infection to the fetus? Select all that apply.

measles mumps rubella

A nurse is conducting a class with group of pregnant women who are all in their first trimester of pregnancy. During the class, the women are discussing the various discomforts that they are experiencing. The nurse would expect to hear reports about which discomforts? Select all that apply.

nausea urinary frequency breast tenderness explanation; Common discomforts associated with the first trimester include nausea, urinary frequency, and breast tenderness. Backache and leg cramps are more common in the second trimester.

A client in the 20th week of pregnancy with a blood pressure of 148/92 mm Hg is diagnosed with mild to moderate hypertension. Which treatment does the nurse anticipate for the client when all testing is negative for kidney and thyroid disease?

none Explanation: Chronic hypertension in the pregnant client is associated with a higher rate of poor pregnancy outcomes, including preterm birth, intrauterine growth restriction, and stillbirth. However, there is no clear benefit to the pregnant client or fetus for treating a client with blood pressures less than 150 mm Hg systolic or less than 100 mm Hg diastolic when testing is negative for kidney or thyroid disease. Clients with severe chronic hypertension (blood pressures equal to or greater than 160 mm Hg systolic and/or equal to or greater than 110 mm Hg diastolic) may be treated with nifedipine or methyldopa. Clients with severe chronic hypertension may also be prescribed aspirin to reduce the risk of developing preeclampsia.

A client with hyperemesis gravidarum is admitted to the facility after being cared for at home without success. What would the nurse expect to include in the client's plan of care?

nothing by mouth

A pregnant client with preeclampsia with severe features has developed HELLP syndrome. In addition to the observations necessary for preeclampsia, what other nursing intervention is critical for this client?

observation for bleeding

A nurse is teaching a client about the reproductive system and the hormones involved. The nurse would include which structures as the source of these hormones? Select all that apply.

ovaries hypothalamus pituitary

Which symptom may indicate a complication (not a common discomfort) in the second or third trimester of pregnancy?

pain underneath the ribs on the right side

A woman with no previous history of heart disease begins to have symptoms of myocardial failure a few weeks before the birth of her first child. Findings include shortness of breath, chest pain, and edema, with her heart also showing enlargement. The nurse suspects which condition?

peripartum cardiomyopathy

A client at 29 weeks' gestation is seen in obstetric triage with reports of heavy vaginal bleeding noted upon waking. The client denies abdominal pain or uterine contractions. Continuous fetal monitoring shows a normal fetal heart rate with no signs of fetal distress. Which is the likely cause of the client's condition?

placenta previa The likely cause of the client's bleeding is a placenta previa, which is usually characterized by painless bleeding. Placental abruption is usually characterized by painful bleeding. Vasa previa is diagnosed via ultrasound and characterized by a velamentous insertion of the umbilical cord or a succenturiate placentation or accessory placenta lobe. Both present a risk for cord compression, which can result in fetal distress. Disseminated intravascular coagulation (DIC) is always the result of another condition and is diagnosed through laboratory testing, not just through objective data (i.e., overt vaginal bleeding).

A woman comes to the clinic reporting her period is late and she is wondering if she is pregnant. Which assessment findings by the nurse would indicate she is exhibiting probable signs of pregnancy? Select all that apply.

positive pregnancy test ballottement softening of the cervix Explanation: Probable signs of pregnancy include a positive pregnancy test, ballottement, and softening of the cervix (Goodell sign). Ultrasound visualization of the fetus, auscultation of a fetal heart beat, and palpation of fetal movements are considered positive signs of pregnancy. Absence of menstruation is a presumptive sign of pregnancy.

During a routine prenatal visit, a client is found to have 1+ proteinuria and a blood pressure rise to 140/90 mm Hg with mild facial edema. The nurse recognizes that the client has which condition?

preeclampsia without severe features

A nurse is providing care to a client being treated for preterm labor with intravenous magnesium sulfate. In the presence of this medication, which assessment should the nurse prioritize?

pregnant parent respirations Because respiratory depression is a sign of magnesium sulfate toxicity, the nurse prioritizes assessment of respirations. Uterine contractions and fetal status are both important assessments for the nurse to make when caring for a client receiving magnesium sulfate. However, the priority assessment is pregnant parent respirations. Headache is a potential side effect of magnesium sulfate and should be assessed and addressed by the nurse, but the priority is respirations.

A client at 7 months' gestation presents to the emergency department with reports of a large gush and continuous leaking of fluid from her vagina. She does have some slight pelvic pressure but denies any contractions. This client is showing clinical signs of which of the following?

premature rupture of membranes The woman with PROM usually presents to the delivery suite with reports of a large gush or continuous leakage of fluid from the vagina. She does not, however, report or present with regular uterine contractions because she is not in labor. Dystocia refers to the slow progression of labor, while precipitous labor refers to labor lasting less than 3 hours.

During the initial prenatal visit, the nurse performs what assessment to guide teaching about nutrition during pregnancy?

prepregnancy BMI

A client presents to the emergency department reporting regular uterine contractions. Examination reveals that her cervix is beginning to efface. The client is in her 36th week of gestation. The nurse interprets the findings as suggesting which condition is occurring?

preterm labor Explanation; Preterm labor is the occurrence of regular uterine contractions accompanied by cervical effacement and dilation (dilatation) before the end of the 37th week of gestation. If not halted, it leads to preterm birth. Normal labor can occur after the 37th week. Dystocia refers to a difficult labor. Precipitate labor is one that is completed in less than 3 hours from the start of contraction to birth.

A client with aortic stenosis is in the 16th week of pregnancy. Which assessment finding(s) indicates to the nurse that the client is experiencing cardiac decompensation? Select all that apply.

productive cough edema of the legs and face respirations 28 breaths/min heart rate 102 beats/min Explanation: The demand for cardiac output increases throughout pregnancy. As the pregnancy progresses, the demands on the heart may increase by as much as 50%, exacerbating any underlying condition that may compromise the heart. By the 8th week of pregnancy, approximately half of the anticipated increase in cardiac output has occurred, which increases the risk for cardiac decompensation. Signs of this decompensation include a productive cough, generalized progressive edema, respiratory rate greater than 25 breaths/min, and a heart rate greater than 100 beats/min. A blood pressure of 118/80 mm Hg is not an indication of cardiac decompensation.

A client at 28 weeks' gestation is admitted for an exacerbation of asthma. Which test(s) will the nurse monitor to determine the severity of the exacerbation? Select all that apply.

pulse oximetry arterial blood gases (ABGs) peak flow measurements Explanation: A client who is admitted with an exacerbation of asthma during pregnancy will have continuous pulse oximetry with a goal of maintaining an oxygen saturation level of 95%. When evaluating the blood gases during an asthma exacerbation in pregnancy, it is important to remember that some respiratory alkalosis is normal in pregnancy. Peak flow is also used to objectively assess the severity of the exacerbation. Hemoglobin and serum carbon dioxide levels are not used to determine the severity of asthma exacerbation in a pregnant client.

A nurse is reviewing a client's history and physical examination findings. Which information would the nurse identify as contributing to the client's risk for an ectopic pregnancy?

recurrent pelvic infections

A woman has presented to the emergency department with symptoms that suggest an ectopic pregnancy. Which finding would lead the nurse to suspect that the fallopian tube has ruptured?

referred shoulder pain

As part of a 31-year-old client's prenatal care, the nurse is assessing immunization history. Which immunization is most relevant to ensuring a healthy fetus?

rubella

The placenta is the site where antibodies in the mother's blood pass into the fetal circulation. These antibodies give passive immunity to the fetus for several common childhood diseases. There are some infections for which the mother does not provide antibodies to the fetus. What infection is the fetus not protected from?

rubella explanation; The fetus does not receive immunity to rubella, cytomegalovirus (CMV), varicella, or measles. If the woman encounters these pathogens during her pregnancy, fetal infection may ensue.

The nurse is concerned that a pregnant client is experiencing abruptio placentae. What did the nurse assess in this client?

sharp fundal pain and discomfort between contractions

A client is concerned about the mercury levels in fish and asks the nurse which fish are safe to eat. The best response is:

shrimp, canned tuna, pollack, and catfish. explanation; The larger the fish, the higher the concentration of mercury will be in that fish. Fish such as shrimp, catfish, anchovies, and sardines are small and therefore have small amounts of mercury.

The nurse is evaluating care provided to a client in the third trimester of pregnancy who has been diagnosed with gestational hypertension. Which finding indicates that treatment has been successful for this client?

urine protein 0

A woman at 12 weeks' gestation comes to the clinic with vaginal bleeding. When assessing the woman further, the nurse would suspect a threatened abortion based on which finding?

slight vaginal bleeding Explanation: Slight vaginal bleeding early in pregnancy, no cervical dilation, and a closed cervical os are associated with a threatened abortion. With an inevitable abortion, passage of the products of conception may occur. No fetal tissue is passed with a threatened abortion

The nurse is discussing sexual health with high school students, and states that the hypothalamus, pituitary gland, and gonads all contribute to hormonal regulation of the female reproductive system. What would be the best information for the nurse to include in the education session about the main effects of the gonadotropic hormones?

stimulation of the formation of ova; stimulation of the secretion of hormones from sex organs; stimulation of the development of secondary female sexual characteristics Explanation: The main effects of LH and FSH include stimulating the formation of ova and stimulating the secretion of hormones from sex organs. Gonadotropic hormones also stimulate the development of secondary female sexual characteristics. The ovaries begin to secrete estrogens. The estrogens include estradiol, estriol, and estrone. After puberty, the corpus luteum of the ovary produces another hormone, progesterone, which functions primarily during pregnancy.

A pregnant woman has arrived to the office reporting vaginal bleeding. Which finding during the assessment would lead the nurse to suspect an inevitable spontaneous abortion (miscarriage)?

strong abdominal cramping

The nurse is caring for a pregnant client who is in her 30th week of gestation and has congenital heart disease. Which finding should the nurse recognize as a symptom of cardiac decompensation with this client?

swelling of the face

The nurse is caring for a female client with subfertility secondary to possible anovulation. The client wants to know what they can do while waiting for an appointment with the health care provider. Which nursing suggestion might the client try while awaiting the appointment?

taking basal body temperature (BBT) each morning before getting out of bed

While triaging messages from the answering machine from clients with the following symptoms, which client would obtain the first visit of the day?

the client at 11 weeks' gestation experiencing abdominal cramping

A nurse has been assigned to four antepartum clients. Which client requires immediate follow-up by the nurse?

the client with a cardiomyopathy whose respiratory rate is 32 breaths/min Explanation: The nurse first assesses the client with the cardiomyopathy whose respirations are 32 breaths/min. A respiratory rate greater than 25 breaths/min is a sign of cardiac decompensation and requires prompt follow-up. A nonreactive nonstress test can be due to a variety of reasons, including medications being taken by the antepartum client and fetal sleep patterns; it requires a continuation of the test for at least 40 minutes or as long as 120 minutes but does not require an immediate follow-up. A client with sickle cell anemia who has hemoglobin of 9.8 g/dl (98 g/l) is considered acceptable. A client with chronic hypertension and a blood pressure of 146/90 mm Hg is considered to have mild disease when untreated or has achieved goals if treated for severe disease.

Most cultures have their own beliefs surrounding pregnancy and prenatal care of the woman. A nurse is assigned to a family that is newly immigrated and alone in the community. The client needs bed rest and help with her activities of daily living. One cultural issue that needs to be assessed is:

the role of the male partner in giving care to his wife.

Implantation generally occurs at which place on the uterus?

the upper posterior surface

A young woman experiencing contractions arrives at the emergency department. After examining her, the nurse learns that the client is at 33 weeks' gestation. What treatment can the nurse expect this client to be prescribed?

tocolytic therapy Tocolytic therapy is most likely prescribed if preterm labor occurs before the 34th week of gestation in an attempt to delay birth and thereby reduce the severity of respiratory distress syndrome and other complications associated with prematurity.

A client with a congenital heart condition had a mechanical valve placed during childhood and is 12 weeks' pregnant. Which care should the nurse anticipate for this client?

twice-monthly visits to a cardiologist

A pregnant woman at term is in the obstetrics unit for induction in the morning. Her membranes rupture, and the external fetal monitor shows deep variable decelerations. The nurse should immediately check the client for:

umbilical cord prolapse. Because the client is not in labor, this development is considered premature rupture of membranes. The sudden onset of deep variable decelerations may indicate umbilical cord prolapse, which is an obstetric emergency that requires immediate intervention.

A nurse is inspecting the external genitalia of a female client. When inspecting the vestibule, the nurse would expect to note openings for which structures? Select all that apply.

urethra vagina Bartholin glands Skene glands

The nurse teaches a primigravida client that lightening occurs about 2 weeks before the onset of labor. What will the mother likely experience at that time?

urinary frequency

The nurse is caring for a client with a 3-week history of hyperemesis gravidarum. The health care provider orders diagnostic testing to rule out the most significant side effect of this condition. What test will the nurse perform?

urine dipstick for ketones Explanation:Hyperemesis gravidarum is a severe form of nausea and vomiting that causes dehydration, electrolyte imbalance and weight loss. The obstetric office nurse is able to identify ketones in the urine, which would indicate that the client's body is breaking down stored fat and protein for cell growth. Of the diagnostic tests identified, the urine dipstick for ketones indicates the most significant result of the hyperemesis. A fingerstick for the blood glucose, especially if symptoms of hypoglycemia are present, would indicate the level of glucose and the need to consume food. Fetal heart sounds monitor the progress of the fetus. Orthostatic blood pressure may be an indication of dehydration but does not provide data as to whether dehydration has led to a breakdown of stored fat and protein.

A nurse is caring for a pregnant client admitted with mild preeclampsia. Which assessment finding should the nurse prioritize?

urine output of less than 15 ml/hr

In OB class, the nursing instructor is discussing the female reproductive anatomy. In order to understand the anatomy and function of the uterus, the instructor explains that there are four sections in the uterus. Which of the following are sections of the uterus? Select all that apply.

uterine isthmus fundus cervix corpus Explanation: The fallopian tubes are attached to the uterus but not a part of it. The other selections are all part of the uterus.

A woman in active labor has just had her membranes ruptured to speed up labor. The nurse is concerned the woman is experiencing a prolapse of the umbilical cord when the nurse notices which pattern on the fetal heart monitor?

variable deceleration pattern Umbilical cord prolapse can be seen after the membranes have ruptured, when the FHR is displaying a sudden variable deceleration FHR pattern on a fetal monitor. It is not uncommon for FHR to increase following a procedure. Early deceleration with each contraction is seen when the fetal head is being compressed through the pelvic opening. Late deceleration with late recovery following contraction is associated with uteroplacental insufficiency (UPI).

A student nurse is assigned to care for a woman who is being admitted for delivery of her baby. Where would the nurse expect to find the urethral meatus?

within the vestibule

A nurse is interviewing a couple at a preconception counseling session. The couple is of Greek heritage and are concerned about the possibility of their children being born with a genetic disorder. Based on the nurse's understanding of genetic disorders, the nurse would identify this couple as being at risk for which condition?

β-thalassemia


Kaugnay na mga set ng pag-aaral

Chp. 5 Temporomandibular Joint (TMJ)

View Set

Chemistry Quiz 2: Precision, Significant Figures, and Scientific Notation

View Set

Chapter 15 Temperature, Heat and Expansion

View Set

Campbell Biology 10th Edition Chapter 30-Plant Diversity II: The Evolution of Seed Plants

View Set

Med-Surg: Cardiovascular Review Questions

View Set

NJ Stopping and Parking Distances

View Set

FINA Exam 2 Conceptual Questions

View Set